LSAT and Law School Admissions Forum

Get expert LSAT preparation and law school admissions advice from PowerScore Test Preparation.

 sekyiste
  • Posts: 6
  • Joined: Nov 25, 2016
|
#30968
Hi!

I got this question right by eliminating the wrong answers but I cant seem to understand why D is the right answer choice? :hmm: An explanation as to why D is right will be greatly appreciated.
User avatar
 Jonathan Evans
PowerScore Staff
  • PowerScore Staff
  • Posts: 726
  • Joined: Jun 09, 2016
|
#30973
Hi, Sekyiste,

This question asks you to "resolve the apparent discrepancy." The first thing you need to do is identify the two parts of the stimulus that don't seem to go together. In other words, which statements in the stimulus appear to conflict with one another? In this question, the conflicting ideas are as follows:
  1. On the one hand, some people in the study who received the Hep A vaccine still ended up exhibiting symptoms of Hep A
  2. On the other hand, the vaccine is completely effective in preventing infection with the Hep A virus
When I'm doing a question like this, after I've identified the two things that don't seem to go together, I ask myself a question:
  • How is it possible that "on the one hand, some people in the study who received the Hep A vaccine still ended up exhibiting symptoms of Hep A" and "on the other hand, the vaccine is completely effective in preventing infection with the Hep A virus?"
How could both these things be true at the same time? What kind of information would help to make this situation make sense.

To prephrase, I try to come up with a hypothetical idea. In this case, I thought, "What if the vaccine itself leads to the appearance of symptoms of Hep A?"

This potential situation would explain how it's possible that "on the one hand, some people in the study who received the Hep A vaccine still ended up exhibiting symptoms of Hep A" and "on the other hand, the vaccine is completely effective in preventing infection with the Hep A virus."

This wasn't what we got in the credited response, but the credited response answers the same question. If we knew that those vaccinated against Hep A had been infected with the virus prior to this vaccination, then it would be possible both for them to develop symptoms and for the vaccine to remain completely effective in preventing new infections.
 Brazilfagan
  • Posts: 3
  • Joined: Feb 28, 2017
|
#33177
Hello! So, I prephase for this question and the answer I got fit answer E. So, I am still confuse on how D is the answer and not E. Could you explain? :-D
User avatar
 Jonathan Evans
PowerScore Staff
  • PowerScore Staff
  • Posts: 726
  • Joined: Jun 09, 2016
|
#33213
Hi, Brazilfagan,

In this Resolve the Discrepancy question, you need to make sure that the answer fits the conditions of the problem.

Just to recap, the apparent conflicting conditions are:
1. On the one hand, some people in the study who received the Hep A vaccine still ended up exhibiting symptoms of Hep A.
2. On the other hand, the vaccine is completely effective in preventing infection with the Hep A virus.
We need to make sure that we pick an answer that accurately addresses each side of this apparent discrepancy.

Answer Choice (E) is not consistent with the information in the stimulus because it states that those who got the vaccine recovered more quickly than those who did not. However, the information in the stimulus indicates that the vaccine is completely effective at preventing infection. Even if the people recovered more quickly, they still would have had to be infected according to answer choice (E). This is not consistent with the stimulus.

Answer Choice (D) shows that the people in the vaccinated group who exhibited symptoms of Hep A already had been infected prior to receiving the vaccine, and this fact addresses both sides of this discrepancy without conflicting with either.

I hope this helps!
 Jerrymakehabit
  • Posts: 52
  • Joined: Jan 28, 2019
|
#63696
Can someone please explain why A is incorrect?

The placebo did not produce side effect resembling the symptoms of hep A. So we can know that the vaccine could have produced side effect resembling the symptoms of hep A. That is why the symptoms of hep A showed up but meanwhile the vaccine is effective. Is it because this inference "So we can know that the vaccine could have produced side effect resembling the symptoms of hep A." could be wrong?

Thanks
Jerry
 Adam Tyson
PowerScore Staff
  • PowerScore Staff
  • Posts: 5153
  • Joined: Apr 14, 2011
|
#63840
I'm not sure I follow your reasoning here, Jerry. If answer A is true, all we know is that the folks in the placebo group did not have any "false positives" for Hepatitis A. That is, the folks who showed symptoms didn't get them from taking the placebo. So, either they actually had the disease, or something other than the placebo caused those symptoms (like maybe another illness that mimics Hep A).

In no way does any of this resolve the paradox in the stimulus. The issue we need to resolve is that the vaccine is completely effective, even though some people who got it showed symptoms. What happened to the placebo group is irrelevant to that paradox! We should be focused completely on the people who got the vaccine. How can they have symptoms if the vaccine completely prevents the disease? Only answer D provides that answer - they already had it, so it was too late to prevent it.

I'll be honest, I was expecting a totally different answer, which was that the vaccine causes some symptoms without the patient actually getting the disease itself. That's actually the case for many vaccines, so it seemed like a natural prephrase to me. But, that wasn't offered as an answer choice, and Answer D was perfect anyway, so it's a winner!

Get the most out of your LSAT Prep Plus subscription.

Analyze and track your performance with our Testing and Analytics Package.